For a linear function y = (1-m) x + m, if M > 1, then the function image does not pass through several quadrants?

For a linear function y = (1-m) x + m, if M > 1, then the function image does not pass through several quadrants?


If M > 1, then 1 - M < 0
So the image goes through quadrants one, two, four
So the image doesn't go through the third quadrant



It is known that the image of the linear function y = (M + 1) x + (M-3) does not pass through the second quadrant
(1) Find the value range of M


When m + 1 = 0, i.e. M = - 1, y = - 4, the image does not pass through the second quadrant, which conforms to the meaning of the question;
When m + 1 ≠ 0, i.e. m ≠ - 1, then k = m + 1 > 0, B = M-3 ≤ 0, so - 1



If the image of the first-order function y = (1m) x + 2m does not pass through the fourth quadrant, then the value range of M?
A;m>0 B: m


∵ the image does not pass through the fourth quadrant;
Then 1m > 0; 2m > 0;
So m > 0;
Choose a



How much is 1 * half + Half * one third +. = it's easy to calculate


1 * half + Half * one third +
=1-1/2+1/2-1/3+1/3-1/4+…… +1/n-1/(n+1)
=1-1/(n+1)
=n/(n+1)



Calculation question √ 1 / 3 √ 12 + √ 24-2 √ 1 / 2,


√ 1 / 3 √ 12 + √ 24-2 √ 1 / 2
=2+2√6-√2



(- 5) × (- half) × 3 × (- 2) × 2
(- 5 / 12) × 8 / 15 × 1 / 2 × (- 2 / 3)


(- 5) × (- half) × 3 × (- 2) × 2
=-5 × half × 3 × 2 × 2
=-(5 × 2) × (half × 2) × 3
=-10×1×3
=-30



Detailed process - 18 △ (- 3) ^ 2 + 5x (negative half) ^ 3 - (- 15) ^ 5


Solution
simple form
=-18÷9+5×(-1/8)+3
=-2-5/8+3
=1-5/8
=3/8



-O. 5 - (- three and a quarter) + 2.75 - (+ seven and a half)


-O. 5 - (- three and a quarter) + 2.75 - (+ seven and a half)
=-0.5+3.25+2.75-7.5
=(3.25+2.75)-(0.5+7.5)
=6-8
=-2



If a = half (√ 5 + √ 3), B = half (√ 5 - √ 3), then what is the square of a - AB + B?


Square of a - AB + square of B = (a-b) square + AB = √ 3 square + 1 / 2 = 3.5



How much is minus 0.5 minus minus two and a quarter plus 2.75 minus seven and a half?
How much is minus 0.5 minus minus two and a quarter plus 2.75 minus seven and a half?


-0.5 - (- 2 and 1 / 4) + 2.75-7 and 1 / 2
=-0.5+2.25+2.75-7.5
=-(0.5+7.5)+(2.25+2.75)
=-8+5
=-3